Download as rtf, pdf, or txt
Download as rtf, pdf, or txt
You are on page 1of 73

41

2) After teaching a client and family about lithium therapy, which of the following client
statements indicates the need for further teaching?

Options:

A) I should drink 10 to 12 glasses of water daily

B) I need to eliminate salt in my diet

C) I should avoid driving until I'm stabilized

D) I'll report any vomiting, diarrhea, blurred vision, or weakness

Correct Answer is: B

Explanation : Clients receiving lithium need to have a consistent dietary intake of sodium to
maintain a therapeutic serum lithium level of 0.6 to 1.2 mEq/L. A decrease in salt intake
decreases lithium elimination, causing an increase in the serum lithium level. The client who is
taking lithium needs to ingest adequate amounts of fluid, at least 2,400 to 3,000 mL (2 1/2 to 3
quarts) per day. Drinking 10 to 12 glasses of water each day would aid in achieving this goal.
Because drowsiness and dizziness can occur with this drug, the client should avoiding driving
until the effects of the drug are known and the client is stabilized. Calling the doctor if vomiting,
diarrhea, blurred vision, and weakness are experienced is important because these symptoms
may indicate lithium toxicity.

5) When preparing the drug teaching plan for a client who is taking phenelzine (Nardil), which of
the following signs and symptoms would the nurse stress as most important to report to the
physician immediately?

Options:

A) Dry mouth

B) Palpitations

C) Urinary hesitancy
D) Blurred vision

Correct Answer is: B

Explanation : Because of the risk for hypertensive crisis with MAOIs such as phenelzine, the
client needs to immediately report palpitations to the physician. This symptom could indicate
hypertension or a hypertensive crisis caused by ingestion of tyramine-rich food while taking the
drug. Although urinary hesitancy, dry mouth, and blurred vision are side effects of phenelzine
and should be reported, immediate reporting of possible signs and symptoms of hypertension or
hypertensive crisis is crucial.

6) The nurse is teaching a client and family about phenelzine (Nardil). Which of the following
foods would the nurse urge the client to avoid?

Options:

A) Sour cream

B) American cheese

C) Chicken

D) Eggs

Correct Answer is: A

Explanation : Because phenelzine is an MAOI, foods high in tyramine need to be avoided to


prevent the development of a hypertensive crisis. Some foods high in tyramine include sour
cream, aged cheeses, yogurt, Chianti, beer, bananas, avocados, salami, sausage, bologna,
caffeinated coffee and colas, and chocolate. High-protein foods that have undergone protein
break-down by aging, fermentation, pickling, or smoking should be avoided. A hypertensive
crisis, evidenced by occipital headache, stiff neck, nausea and vomiting, sweating, nosebleed,
dilated pupils, tachycardia, and constricting chest pain, can occur with this food-drug
combination.

7) A 67-year-old client will be discharged to home with imipramine (Tofranil). Which of the
following would be most important to include when instructing the client and spouse about the
medication?
Options:

A) Eat a high-fiber diet

B) Avoid the ingestion of alcohol

C) Urinate as soon as the urge is felt

D) Wear sunglasses outdoors

Correct Answer is: B

Explanation : Alcohol potentiates the central nervous system depression that can occur with
imipramine, leading to increased sedation, confusion, and disorientation, and consequently
placing the client at risk for injury. Therefore, instructing the client and spouse about avoiding
alcohol is most important.

9) A client whose symptoms of schizophrenia are under control with olanzapine (Zyprexa) and
who is functioning at home and in her part-time employment, states that she is very concerned
about her 20-pound weight gain since she started taking the medication 6 months ago. Which of
the following responses by the nurse would be most appropriate?

Options:

A) Discuss nutrition, daily diet, and exercise with the client

B) Advise the client to decrease her dosage by one-half

C) Tell the client not to worry because she should stop gaining weight

D) Suggest that the client talk with her physician about changing to another antipsychotic

Correct Answer is: A

Explanation : The nurse would discuss nutrition, daily diet, and exercise with the client
concerned about her weight gain while taking olanzapine (Zyprexa). Weight gain is common with
this drug therapy. The client would benefit from nutrition and exercise teaching, and the nurse
should provide the client with an initial course of action. Suggesting that the client talk with her
physician about changing to another antipsychotic may not be in her best interest, because
olanzapine (Zyprexa) is keeping the symptoms of her illness undercontorl and she is able to
function at home and on the job. Advising her to cut her dose in half may lead to
decompensation. Telling her not to worry because she should stop gaining weight minimizes the
client's concern. Also, additional weight gain is possible.

10) A client taking risperidone (Risperdal) 2 mg PO twice daily informs the clinic nurse that she
will be getting married in 3 months to another client she met at the outpatient clinic. During the
client interview, which of the following would be an area of priority concern?

Options:

A) The possibility of or plan for pregnancy

B) Living arrangements

C) Money management

D) Her fiance's medication compliance

Correct Answer is: A

Explanation : The nurse would be most concerned about the possibility of or plan for pregnancy
for a male for a female client receiving antipsychotic medication. Most antipsychotic medications
are contraindicated during pregnancy because of potential injury to the fetus.

11) Assessment for which of the following side effects would be the priority for a client who is
taking risperidone (Risperdal) 1 mg PO twice daily?

Options:

A) Anxiety

B) Postural hypotension

C) Headache

D) Insomnia

Correct Answer is: B

Explanation : Significant postural hypotension is associated with risperidone (Risperdal) therapy.


The nurse would monitor the client's blood pressure sitting and standing and teach the client
interventions to manage this side effect to prevent risk for injury. Although insomnia, headache,
and anxiety are possible side effects of risperidone therapy, they are of less immediate concern
than postural hypotension.

14) Which of the following would the nurse include in a teaching plan that addresses the side
effects of antipsychotic medication?

Options:

A) Percentages associated with each side effect

B) Side effects that can be seen or felt

C) Information about all potential side effects

D) Research data about rare side effects

Correct Answer is: B

Explanation : The nurse needs to focus on side effects that can be seen or felt, using a simple,
brief, written description of the benefits of the medication and a list of common side effects and
how to cope with them. The written format helps the client and family feel more in control by
Participating in treatment. They also can use the written information as a helpful resource of
review. Information about all potential side effects, including percentages associated with each,
will cause undue anxiety in the client and possibly overwhelm the client and family, negatively
affecting compliance. The nurse should use discretion in selecting the content of educational
sessions.

15) Which of the following would the nurse include when teaching the family and a client who
was prescribed benztropine (Cogentin), 1 mg PO twice daily, about the drug therapy?

Options:

A) The drug can be used with over-the-counter cough and cold preparations

B) Antacids can be used freely when taking this drug

C) The client should not discontinue taking the drug abruptly

D) Alcohol consumption with benztropine therapy need not be restricted

Correct Answer is: C


Explanation : The client should not discontinue taking the drug abruptly

16) Which of the following medications would the nurse expect to administer to a client who is
experiencing an oculogyric crisis?

Options:

A) Chlorpromazine (Thorazine) 50 mg

B) Thioridazine (Mellaril) 100 mg

C) Procyclidine (Kemadrin) 5 mg

D) Benztropine (Cogentin) 1 mg

Correct Answer is: D

Explanation : An oculogyric crisis is a severe dystonic reaction typically caused by the older
generation of antipsychotics. The nurse would administer 1 to 2 mg of Benztropine (Cogentin)
intramuscularly to provide a prompt onset of action and offer the client reassurance. Cogentin is
generally used to threat drug-induced extrapyramidal side effects (EPSEs). Chlorpromazine and
thioridazine (Mellaril) are traditional antipsychotic agents that would intensify the client's
oculogyric crisis. Although procyclidine (Kemadrin) is effective for treating the rigidity and
sialorrhea associated with antipsychotics, it is available only in oral form and would not be used
in a crisis, when a parenteral form is needed for quicker onset of action.

17) Unresolved feelings related to loss most likely may be recognized during which phase of the
therapeutic nurse-client relationship?

Options:

A) Orientation

B) Working

C) Termination

D) Trusting

Correct Answer is: C


Explanation : In the termination phase the relationship comes to a close.Ending treatment
sometimes may be traumatic for clients who have come to value the relationship and the
help.Because loss is an issue,any unresolved feelings related to loss may resurface during this
phase.

22) A client admitted to the mental health unit is experiencing disturbed thought processes.The
client believes that the food is being poisoned.which communication technique does the nurse
plan to use to encourage the client to eat?

Options:

A) Using open-ended questions and silence.

B) Offering opinions about the necessity of adequate nutrition

C) Identifying the reasons that the client may not want to eat.

D) Focusing on self-disclosure regarding food preferences

Correct Answer is: A

Explanation : Open-ended questions and silence are strategies used to encourage clients to
discuss their problem.Option 2 and 3 are not helpful to the client because they do not encourage
the client to express feelings.The nurse should not offer opinions and should encourage the
client to identify the reasons for the behaviour.Option 4 is not a client centered intervention.

23) Unresolved feelings related to loss most likely may be recognized during which phase of the
therapeutic nurse-client relationship?

Options:

A) Trusting

B) Termination

C) Working

D) Orientation

Correct Answer is: B


Explanation : The characteristics of termination phase is unresolved feelings related to loss.

28) A client admitted to the mental health unit is experiencing Disturbed Thought Process.The
client believes that the food is being poisoned.Which communication technique does the nurse
plan to use to encourage the client to eat?

Options:

A) Focusing on self-disclosure regarding food preferences

B) Using open-ended questions and silence

C) Offering opinions about the necessity of adequate nutrition

D) Identifying the reasons that the client may not want to eat

Correct Answer is: B

Explanation : Using open-ended questions and silence will allow them to discuss more about
their thoughts.

29) The nurse is working with a client who has sought counselling after trying to rescue a
neighbor involved in a house fire.In spite of the clients efforts,the neighbor died.Which action
does the nurse engage in with the client during the working phase of the nurse-client
relationship?

Options:

A) Explorinfg the clients ability to function

B) Exploring the clients potential for self-harm

C) Inquiring about and examining the clients feelings that may block adaptive coping

D) Inquiring about the clients perception or appraisal of the neighbours

Correct Answer is: C

Explanation : Working phase of the nurse-client relationship includes inquiring about and
examining the clients feelings that may block adaptive coping.
30) A client who has just been sexually assaulted in quiet and calm.The nurse analyzes this
behaviour as indicating which defense mechanism?

Options:

A) Rationalization

B) Denial

C) Projection

D) Intellectualization

Correct Answer is: B

Explanation : Denial is used here by the client.

31) The nurse completes the initial assessment of a client admitted to the mental health unit.The
nurse analyzes the data obtained on assessment and determines that which of the following
presents a priority concern?

Options:

A) The presence of bruises on the clients body

B) The significant others disapproving of the treatment

C) The clients report of not eating or sleeping

D) The clients report of suicidal thoughts

Correct Answer is: D

Explanation : Comparing other concerns reporting of suicidal thoughts is at the first priority as it
has the severe consequence.

33) An inebriated client is brought to the emergency department by the local police.The client is
told that the physician will be in to see the client in about 30 minutes.the client becomes loud
and offensive and wants to be seen by the physician immediately.The most appropriate nursing
intervention is which of the following?

Options:

A) Attempt to talk with the client to deescalate behavior

B) Inform the client that the client will be asked to leave if the behavior continues

C) Offer to take the client to an examination room until the client can be treated

D) Watch the behavior escalate before intervening

Correct Answer is: C

Explanation : Offer to take the client to an examination room until the client can be treated.

36) The supervisor reprimands the nurse in charge of the nursing has not adhered to the unit
budget.Later that afternoon,the charge nurse accuses the nursing staff of wasting supplies.The
behavior is an example of

Options:

A) Denial

B) Displacement

C) Repression

D) Suppression

Correct Answer is: B

Explanation : Displacement

49) The nurse is preparing to provide reminiscence therapy for a group of clients.Which of the
following clients would the nurse select for this group?

Options:

A) A client with mild depression who demonstrates normal cognition

B) An undifferentiated schizophrenic client with moderate cognitive impairment


C) A catatonic,immobile client with moderate cognitive inpairment

D) A client who exhibits profound depression with moderate cognitive impairment

Correct Answer is: A

Explanation : Reminiscence therapy is for mild depression clients who demonstrates normal
cognition.

51) The client asks the nurse about milieu therapy.The nurse responds knowing that the the
primary focus of milieu therapy can best be described as which of the following?

Options:

A) A behavioral approach to changing behavior

B) A living,learning ,or working environment

C) A cognitive approach to changing behavior

D) A form of behavior modification therapy

Correct Answer is: B

Explanation : The primary focus of milieu therapy is about living,learning ,or working
environment

52) The nurse is caring for a client with a phobia who is being treated for the condition.The client
is introduced to short periods of exposure to the phobic object while in a relaxed state.The nurse
understands that this form of behavior modification can best be described as

Options:

A) Aversion therapy

B) Self-control therapy

C) Mileu therapy

D) Systematic desensitization
Correct Answer is: D

Explanation : This form of behavior modification can best be described as Systematic


desensitization.

53) A client with an eating disorder is planning to attend group meetings with Overeaters
Anonymous, and the nurse describes this group to the client.The nurse determines this group to
the client.The nurse determines that the client needs additional information if the client states
which of the followings about this self-help group?

Options:

A) The members provide support to each other

B) People who have a similar problem are able to help others

C) The leader is a nurse or psychiatrist

D) It is designed to serve people who have a common problem

Correct Answer is: C

Explanation : In group meetings the members of the group provide support to each other,People
who have a similar problem are able to help others,It is designed to serve people who have a
common problem.It should not necessary the leader should be a nurse or psychiatrist

54) The client is preparing to attend a Gamblers Anonymous meeting for the first time.The
prototype used by this group is the 12-step program developed by Alchoholics Anonymous.The
nurse tells the client that the first step in the 12-step program is which of the following?

Options:

A) Discontinuing relationships with friends who are gamblers

B) Stating that the gambling will be stopped

C) Admitting to having a problem

D) Substituting gambling for other activities


Correct Answer is: C

Explanation : The first step in the 12-step program is admitting to having a problem.

55) The nurse is conducting a group therapy session,and a client with a manic disorder is
monopolizing the group.The most appropriate nursing actions is which of the following?

Options:

A) Tell the client to stop monopolizing the group

B) Ask the client to leave

C) Suggest that the client stop talking and try listening to others

D) Refer the client to another group

Correct Answer is: C

Explanation : When a nurse conducts the group session the appropriate action is suggest the
client stop talking and try listening to others which helps them to understand.

56) The nurse is planning to formulate a psychotherapy group.Several clients are interested in
attending the session.The nurse plans the group,knowing that the maximum number of group
members to include in this group is

Options:

A) 12

B) 10 .

C) 16

D) 14

Correct Answer is: B

Explanation : The maximum number of psychotherapy group is 10.


57) A nurse employed in a mental health unit of a hospital is the leader of a group
psychotherapy session.The nurses role in the termination stage of each group development is to

Options:

A) Encourage problem-solving

B) Acknowledge the contributions of each group member

C) Encourage accomplishment of the groups work

D) Encourage members to become acquainted with one another

Correct Answer is: B

Explanation : The nurses role in the termination stage of each group development is to
acknowledge the contributions of each group member.

58) A 60-year-old client has a maintenance lithium level of 0.6 mEq/L. Which of the following
would the nurse expect the client to exhibit?

Options:

A) Fairly good control of symptoms of mania

B) Manifestations of acute mania

C) A decrease in manic symptoms

D) Signs and symptoms of lithium toxicity

Correct Answer is: A

Explanation : Maintenance serum levels between 0.4and 0.8 mEq/L are considered appropriate
and therapeutic for elderly clients. Therefore, the nurse would expect fairly good control of
symptoms of mania with a serum level of 0.6 mEq/L. Signs and symptoms of lithium toxicity
would be evidenced with a serum level that is greater than 1.5 mEq/L. Manifestations of acute
mania would suggest nontherapeutic serum levels of lithium, as indicated by a serum level of
less than 0.4 mEq/L. Manic symptoms abate as the lithium level increases. Typically it takes 7 to
10 days to achieve a clinical response.
62) The statement that would best describe the practice of psychiatric nursing would be

Options:

A) Ensuring client's legal and ethical rights by acting as a client advocate

B) Helping people with present or potential mental health problems

C) Focusing interpersonal skills on people with physical or emotional problems

D) Acting in a therapeutic way with people who are diagnosed as having a mental disorder

Correct Answer is: B

Explanation : An important aspect of the role of the psychiatric nurse is primary, secondary, and
tertiary interventions to promote emotional equilibrium.

63) For most nurses the most difficult part of the nurse-client relationship is

Options:

A) Developing an awareness of self and the professional role in the relationship

B) Accepting responsibility in identifying and evaluating the real needs of the client

C) Being able to understand and accept the client's behavior

D) Remaining therapeutic and professional at all times

Correct Answer is: A

Explanation : The nurse's major tool in psychiatric nursing is the therapeutic use of self.
Psychiatric nurses must learn to be aware of their own feelings and how they affect the situation.

64) In psychiatric nursing, the most important tool the nurse brings to a helping relationship is

Options:

A) Years of experience in milieu management

B) Knowledge of psychopathology
C) Oneself and a desire to help

D) Advanced communication skills

Correct Answer is: C

Explanation : The nurse brings to a therapeutic relationship the understanding of self and basic
principles of therapeutic communication; this is the unique aspect of the helping relationship.

67) Which of the following factors would best support a diagnosis of substance abuse?

Options:

A) Interference with activities of daily living

B) Legal complications related to substance use

C) Daily substance use

D) Spending money

Correct Answer is: A

Explanation : Drug abuse is the maladaptive and consistent use of a drug despite social,
occupational, psychological or physical problems exacerbated by the drug.

68) Which of the following describes a person using words that have no known meaning?

Options:

A) Neologisms

B) Neolithic

C) Verbalism

D) Delusional blocking

Correct Answer is: A


Explanation : Neologisms describes A person using words that have no known meaning.

69) Which of the following categories would a 20 year old adult be placed in?

Options:

A) Intimacy vs. Isolation

B) Generativitiy vs. Stagnation

C) Integrity vs. Despair

D) Longevity vs. Guilt

Correct Answer is: A

Explanation : Intimacy vs. Isolation

70) Which of the following categories would a 60 year old adult be placed in?

Options:

A) Intimacy vs. Isolation

B) Generativitiy vs. Stagnation

C) Integrity vs. Despair

D) Longevity vs. Guilt

Correct Answer is: B

Explanation : 60 year old adult be placed in Generativitiy vs. Stagnation

71) Which of the following categories would a 70 year old adult be placed in?

Options:

A) Intimacy vs. Isolation


B) Generativitiy vs. Stagnation

C) Integrity vs. Despair

D) Longevity vs. Guilt

Correct Answer is: C

Explanation : 70 year old adult be placed in Integrity vs. Despair.

72) Which of the following is not a characteristic of a panic disorder?

Options:

A) Nausea

B) Excessive perspiration

C) Urination

D) Chest pain

Correct Answer is: C

Explanation : Characteristics of Panic disorder are Nausea, Excessive perspiration , Chest pain.

73) Which of the following best describes a person that is unable to tell you were there hand or
foot is?

Options:

A) Autotopagnosia

B) Cataplexy

C) Ergophobia

D) Anosognosia

Correct Answer is: A


Explanation : Autotopagnosia is agnosia that affects the sense of posture. It is characterized by
an inability to localize and orient different parts of the body. Cataplexy is a sudden and transient
episode of loss of muscle tone, often triggered by emotions but frequently affects people who
have narcolepsy, a disorder whose principal signs are EDS (Excessive Daytime Sleepiness), sleep
attacks, sleep paralysis, hypnagogic hallucinations and disturbed night-time sleep. Ergophobia
also called Ergasiophobia, is an abnormal and persistent fear (or phobia) of work, finding work or
functioning, ergophobia may also be a subset of either social phobia or performance anxiety.
Anosognosia is a condition in which a person who suffers disability seems unaware of or denies
the existence of his or her disability. This may include unawareness of quite dramatic
impairments, such as blindness or paralysis.

74) Which of the following best describes a person that is completely awake falling asleep
spontaneously?

Options:

A) Cataplexy

B) Narcolepsy

C) Transitional sleep

D) REM absence

Correct Answer is: A

Explanation : Cataplexy is a sudden and transient episode of loss of muscle tone, often triggered
by emotions but frequently affects people who have narcolepsy, a disorder whose principal signs
are EDS (Excessive Daytime Sleepiness), sleep attacks, sleep paralysis, hypnagogic hallucinations
and disturbed night-time sleep

75) Which of the following is not a sign of anxiety?

Options:

A) Dyspnea

B) Hyperventilation

C) Moist mouth

D) GI symptoms
Correct Answer is: C

Explanation : Dyspnea, Hyperventilation, GI symptoms all are signs of anxiety except moist
mouth.

76) Parallel play for toddlers develops in this age range?

Options:

A) 5-10 months

B) 10-14 months

C) 12-24 months

D) 24-48 months

Correct Answer is: D

Explanation : Parallel play for toddlers develops during 24-48 months of age.

77) Which of the following reflexes is not found at birth?

Options:

A) Babinski

B) Palmar

C) moro

D) Flexion

Correct Answer is: D

Explanation : Except flexion all are found at the time of birth.

78) The REM sleep cycle occur approximately every ____ minutes?
Options:

A) 45

B) 60

C) 75

D) 90

Correct Answer is: D

Explanation : The REM sleep cycle occur approximately every 90 minutes.

79) Which of the following waveforms is most commonly found when you are awake?

Options:

A) Theta

B) Alpha

C) Beta

D) Zeta

Correct Answer is: C

Explanation : When we are awake Beta wave forms will be found in the graph.

80) Which of the following matches the definition: response to severe emotion stress resulting in
involuntary disturbance of physical functions?

Options:

A) Conversion disorder

B) Depressive reaction

C) Bipolar disorder

D) Alzheimers disease
Correct Answer is: A

Explanation : Response to severe emotion stress resulting in involuntary disturbance of physical


functions is a Conversion disorder.

81) Which of the following matches the definition: the justification of behaviors using reason
other than the real reason?

Options:

A) Compensation

B) Projection

C) Rationalization

D) Dysphoria

Correct Answer is: C

Explanation : Compensation - Putting forth extra effort to achieve in areas where one has a real
or imagined deficiency. Projection - Transferring ones internal feelings, thoughts, and
unacceptable ideas and traits to someone else. Dysphoria - a state of feeling unwell or unhappy.
Rationalization -An attempt to make unacceptable feelings and behavior acceptable by justifying
the behavior.

82) Which of the following matches the definition: attributing of our own unwanted trait onto
another person?

Options:

A) Compensation.

B) Projection

C) Rationalization

D) Dysphoria
Correct Answer is: B

Explanation : Compensation - Putting forth extra effort to achieve in areas where one has a real
or imagined deficiency. Projection - Transferring ones internal feelings, thoughts, and
unacceptable ideas and traits to someone else. Dysphoria - a state of feeling unwell or unhappy.
Rationalization -An attempt to make unacceptable feelings and behavior acceptable by justifying
the behavior.

83) Object permanence for toddlers develops in this age range?

Options:

A) 5-10 months

B) 10-14 months

C) 12-24 months

D) 15-24 months

Correct Answer is: C

Explanation : Object permanence for toddlers develops during 12-24 months.Refer Growth and
development of the child.

84) Which of the following months matches with an infant first having the ability to sit-up
independently?

Options:

A) 4 months

B) 6 months

C) 8 months

D) 10 months

Correct Answer is: B

Explanation : Refer Growth and development of the child.In six months an infant will have the
ability to sit-up independently.

85) Which of the following matches the definition: covering up a weakness by stressing a
desirable or stronger trait?

Options:

A) Compensation

B) Projection

C) Rationalization

D) Dysphoria

Correct Answer is: A

Explanation : Projection - Transferring ones internal feelings, thoughts, and unacceptable ideas
and traits to someone else. Dysphoria - a state of feeling unwell or unhappy. Rationalization -An
attempt to make unacceptable feelings and behavior acceptable by justifying the behavior.

88) A 93 year-old female with a history of Alzheimer’s Disease gets admitted to an Alzheimer’s
unit. The patient has exhibited signs of increased confusion and limited stability with gait.
Moreover, the patient is refusing to use a w/c. Which of the following is the most appropriate
course of action for the nurse?

Options:

A) Recommend the patient remain in her room at all times

B) Recommend family members bring pictures to the patient’s room

C) Recommend a speech therapy consult to the doctor

D) Recommend the patient attempt to walk pushing the w/c for safety

Correct Answer is: B

Explanation : Stimulation in the form of pictures may decrease signs of confusion.


89) A 64 year-old Alzheimer’s patient has exhibited excessive cognitive decline resulting in
harmful behaviors. The physician orders restraints to be placed on the patient. Which of the
following is the appropriate procedure?

Options:

A) Secure the restraints to the bed rails on all extremities.

B) Notify the physician that restraints have been placed properly.

C) Communicate with the patient and family the need for restraints.

D) Position the head of the bed at a 45 degree angle.

Correct Answer is: C

Explanation : Both the family and the patient should have the need for restraints explained to
them.

90) A 22 year-old patient in a mental health lock-down unit under suicide watch appears happy
about being discharged. Which of the following is probably happening?

Options:

A) The patient is excited about being around family again.

B) The patient’s suicide plan has probably progressed.

C) The patient’s plans for the future have been clarified.

D) The patient’s mood is improving

Correct Answer is: B

Explanation : The suicide plan may have been decided.

91) What is the best approach to an anti-social personality?

Options:

A) ignoring the patient


B) allowing the patient to do whatever he or she likes

C) forbidding the patient from doing anything that he or she likes

D) setting limits on behaviors

Correct Answer is: D

Explanation : Individuals with anti-social personality disorder need firm boundaries on their
behavior

92) When informed consent is obtained from the patient, the explanation of the surgical
procedure, possible risks, complications and alternatives is the responsibility of the

Options:

A) admission clerk.

B) surgeon.

C) registered nurse.

D) anesthesiologist.

Correct Answer is: B

Explanation : The surgeon must explain the procedure in terms that the patient or surrogate is
able to understand. It is the surgeon who is responsible to explain the surgical procedure to the
patient.

93) The best nursing approach to parents who are displaying anxiety and guilt when their child is
hospitalized is to

Options:

A) distract their attention to something less painful.

B) anticipate their emotional responses and acknowledge them.

C) explain the dangers of excess anxiety and guilt.


D) give personal examples that are similar to their situation.

Correct Answer is: B

Explanation : Parents may have a wide range of responses to their child's illness and
hospitalization. Following the diagnosis of an illness, they may react with disbelief, anger or guilt.
Parents tend to search for self-blame regarding why the child became ill or to project anger at
others for some wrongdoing. Fear, anxiety and frustration are common feelings expressed by
parents when their child is hospitalized. The best nursing approach to parents is for the nurse to
anticipate and recognize these responses and allow parents identify the reasons for their
feelings and emphasizing that each is normal, expected and a healthy response to stress
provides them with the opportunity to lessen their emotional burden.

94) To which of the following nursing diagnoses would a nurse give priority when caring for a
patient who is not eating or bathing and refuses to leave the house since the death of her
daughter a year ago?

Options:

A) Dysfunctional grieving related to loss

B) Anticipatory grieving related to fear of social interaction

C) Anxiety related to fear of death

D) Self-care deficit related to loss

Correct Answer is: A

Explanation : Dysfunctional grieving related to loss is the priority nursing diagnosis. This
diagnosis includes the presenting symptoms of isolation, self-neglect and social withdrawal.

95) While orienting a new nurse to the unit, the charge nurse stresses the importance of
accurate documentation. The primary reason for a nurse to document care accurately is to

Options:

A) facilitate insurance reimbursement.

B) be in compliance with individual regulatory agencies.


C) demonstrate responsibility and accountability.

D) prevent any legal action against the healthcare facility and its staff.

Correct Answer is: C

Explanation : Documentation is defined as anything written or printed that is relied on as a


record of proof for authorized persons. Effective documentation reflects the quality of care and
provides evidence of each healthcare team member's accountability in giving care.
Accountability means that the nurse is responsible professionally and legally for the type and
quality of nursing care provided.

96) An 80-year-old with right-sided weakness following a cerebrovascular accident (CVA) is to


ambulate with the aid of a walker. What age-related changes in this individual will increase the
risk of injury?

Options:

A) Slowed response to sensory stimuli

B) Improved muscle mass and strength

C) Development of arcus senilis

D) Increased cognitive functioning

Correct Answer is: A

Explanation : As age increases they will experience Slowed response to sensory stimuli.And this
increases the risk of injury.

97) A nurse planning an educational program for families of patients who have attempted suicide
should include which of the following statements in his teachings?

Options:

A) Suicidal patients will often completely isolate themselves immediately prior to the attempt.

B) Suicidal patients display subtle changes in behavior.

C) Suicidal patients do not display any changes that might indicate their despair.
D) Suicidal patients usually give out many overt clues.

Correct Answer is: B

Explanation : Suicidal patients give clues about their intentions. Very subtle clues may be ignored
or disregarded by others. These can be verbal statements about problems being solved or
behaviors, such as putting personal affairs in order.

98) Which of the following risk factors should a nurse recognize as the most reliable predictor of
patient violence?

Options:

A) Pacing behavior

B) Family history of violence

C) Past history of violence

D) Verbal threats

Correct Answer is: C

Explanation : A past history of violence is the most reliable predictor of patient violence in the
assessment of risk factors. Individuals who have used violence in the past as a method to cope
with stress are more likely to resort to the use of aggressive behavior in the future.

99) Under the Good Samaritan Act, a nurse may be held liable for patient abandonment at the
scene of an emergency in which of the following cases?

Options:

A) The nurse does not stop to provide assistance.

B) The nurse does not initiate care.

C) The nurse begins assitance and then abruptly stops.

D) The nurse does not perform under the direct order of a physician.
Correct Answer is: C

Explanation : If a nurse chooses to stop and give aid at the scene of an emergency, the Good
Samaritan Act provides the following guidelines. The nurse should give care that any reasonable,
prudent person would consider first aid. Do not do what you don't know. Offer assistance; do not
insist. Do not leave the scene until the injured victim leaves or another qualified person takes
over.

100) During a family therapy session, the nurse notes that the wife is sitting with her arms and
legs crossed and her body turned away from her husband. This non-verbal behavior is an
example of

Options:

A) incongruence.

B) blocking.

C) distancing.

D) cultural posturing.

Correct Answer is: C

Explanation : Crossed arms and legs and turning away from a person are nonverbal behavioral
signs that convey an unwillingness to communicate with another. The use of this closed body
language is one way the wife conveys that she is distancing herself from a meaningful
conversation.

103) Which of the following nursing diagnoses would be appropriate for a patient who has a
phobia?

Options:

A) Sensory perceptual alteration

B) Fear related to specific stimulus

C) Self-esteem disturbance

D) Alteration in thought process


Correct Answer is: B

Explanation : The most appropriate diagnosis for a patient who has a phobia is fear related to
specific stimulus. The essential feature of a phobia is a marked, persistent and excessive or
unreasonable fear when in the presence of, or when anticipating an encounter with, a specific
object or situation. The individual recognizes the fear as excessive or unreasonable.

104) To provide psychiatric home care to a patient, which of the following information would be
most important for the nurse to obtain first?

Options:

A) Written orders from a phychiatrist

B) A chronic mental illness diagnosis

C) Home-bound status

D) Payment source

Correct Answer is: A

Explanation : The nurse should obtain written orders from a psychiatrist before providing home
care to a psychiatric patient. The nurse should know the reason for home care and the
medications the patient is to receive before a visit is made.

106) After the completion of an incident report, the nurse places the document in the patient's
chart. The nurse should understand that incident reports

Options:

A) are maintained by the hospital and used as a staff evaluation tool.

B) are not considered legal documents but rather a risk management tool.

C) should be reviewed by the patient poor to discharge.

D) will prevent legal action against the nurse.


Correct Answer is: B

Explanation : Nurses are legally bound to report critical incidents to nursing management. A
form called an incident report (or an unusual occurrence report) is completed and forwarded to
nursing administration as well as hospital or agency administration. The document is used as a
risk management tool but does not become part of the patient's legal document. (i.e., the
patient chart).

107) In which of the following ways should the nurse intervene when a patient repeatedly talks
about the past?

Options:

A) Ask the patient to think of recent pleasures

B) Encourage the patient to share memories

C) Help the patient to establish goals for the future

D) Give the patient a diversional activity

Correct Answer is: B

Explanation : Encouraging the patient to share memories can provide a way of working through
unresolved issues from the past. The nurse can validate the patient's feelings and help the
patient come to terms with painful issues. This helps the patient attain a sense of positive
identity.

108) When discussing methods of stress reduction with a patient, the nurse should use which of
the following approaches first?

Options:

A) Determine if the patient is able to identify sources of stress

B) Teach the patient relaxation techniques that reduce stress

C) Explain to the patient the physiological effects of stress

D) Describe to the patient the benefits of active exercise in coping with stress
Correct Answer is: A

Explanation : The patient should be able to identify signs and symptoms of stress or anxiety so
that the techniques of stress reduction can be used effectively.

109) The most common adverse effect of electroconvulsive therapy (ECT) for which the nurse
must plain interventions is

Options:

A) severe hypertension.

B) physical injury.

C) memory loss.

D) arrhythmia.

Correct Answer is: C

Explanation : Confusion and memory loss are the most common side effects of electroconvulsive
therapy (ECT). The memory loss and confusion increase as the number of treatments increases.
The nurse should plan interventions that address appropriate safety measures.

112) The most important factor the registered nurse should consider when delegating patient
care to a nursing assistant is

Options:

A) the level of skill needed to care for the patient.

B) the nursing assistant's ability to communicate with other members of the health team.

C) the nursing assistant's response to being observed by a registered nurse.

D) the individual's competence with patient assessment.

Correct Answer is: A

Explanation : The registered nurse should know the level of skill required to care for a patient so
that the nurse can assign appropriate staff who can best meet the patient's needs.
114) A woman who has been identified as a victim of domestic violence is seen by the
community mental health nurse. The priority outcome is for the client to

Options:

A) accept responsibility for her role in the abusive relationship.

B) develop conflict resolution skills.

C) explore her relationship with her parents.

D) make a personal safety plan.

Correct Answer is: C

Explanation : The patient's response to this question will demonstrate her ability to problem-
solve, her impulse control and her coping strategies. It would provide the most information on
readiness for discharge.

119) A patient is admitted involuntarily to the psychiatric unit. Which of the following patient
actions would have indicated the need for involuntary rather than voluntary admission?

Options:

A) The patient made threatening remarks to his neighbor.

B) The patient tried to harm his wife.

C) The patient stated that voice were telling him to end his life.

D) The patient tried to stop traffic outside his home.

Correct Answer is: B

Explanation : A patient may be admitted involuntarily to a psychiatric unit only when he/she tries
to harm self or others.

120) Which of the following types of activity would be therapeutic for a patient who is acutely
psychotic?
Options:

A) Task-oriented

B) Competitive

C) Creative

D) Independent

Correct Answer is: A

Explanation : The nurse should assign solitary, non-competitive activities that take some
concentration. Activities that demand concentration keep the patient's attention on reality and
minimize hallucinatory and delusional preoccupation.

121) Which of the following assessment criteria would directly affect the mental health needs of
a community?

Options:

A) The opinions of the residents

B) The demographics of the community

C) Recreational facilities

D) Religious affiliations

Correct Answer is: B

Explanation : Studying the demographic of the community will determine what types of
treatment modalities are most needed in the community. For example, if there are a large
number of elderly living in the area, a geriatric center that offers outpatient, day and inpatient
treatment programs might be established.

123) Which of the following observations would be most significant when assessing the parents
of a child who is suspected of being physically abused?

Options:
A) The parents appear distraught and upset when asked about the injuries.

B) The parents give a history of the injuries that is not compatible with the actual injuries.

C) The parents cannot recall when the last series of immunizations was given.

D) The parents seem eager to take the child home as soon as possible.

Correct Answer is: B

Explanation : Incompatibility between the history and the injury is probably the most important
criterion on which to base the decision to report suspected abuse.

124) When assessing the psychiatric patient, a nurse should recognize which of the following
strengths as essential to successful living?

Options:

A) Ability to drive

B) Ability to work

C) Social skills

D) Knowledge about medications.

Correct Answer is: C

Explanation : Social skills consist of simple interactions such as introducing one's self, starting
and ending a conversation and asking for help. The patient must be able to effectively interact to
get basic needs met.

125) Which of the following actions should the nurse take first following a violent episode on a
psychiatric unit?

Options:

A) Contact hospital administration

B) Conduct a staff debriefing


C) Discuss the incident with the other patients

D) Call hospital security

Correct Answer is: B

Explanation : After the crisis (violent episode) is over, it is recommended that the team discuss
any concerns they may have had during the crisis, since this type of occurrence can be stressful
for staff as well as patients.

126) A patient makes sexually inappropriate comments to the nurse. Which of the following
measures would most likely prevent such behavior?

Options:

A) Refrain from being alone with the patient

B) Clarify nurse-patient roles with the patient

C) Assign a staff member of the same gender to care for the patient

D) Avoid sexual topics of discussion with the patient

Correct Answer is: B

Explanation : Frequently restating the nurse's role throughout the relationship can help the
patient to maintain boundaries

127) A patient who has mental retardation is admitted to a general psychiatric unit. Which of the
following actions should the nurse include initially in the patient's plan of care?

Options:

A) Involve the patient in cognitive restructuring

B) Assess the patient's level of functioning

C) Provide reality orientation

D) Encourage participation in the existing program


Correct Answer is: B

Explanation : Assessing several areas of functioning such as intellectual functioning, activities of


daily living and coping mechanisms helps the nurse to fully develop the plan of care.

128) Several patients have reported to the charge nurse that one of the nurses doesn't come
when called and is very grouchy and ill-humored. The charge nurse knows that the nurse is
having personal problems. The charge nurse's best initial action would be to

Options:

A) report these complaints to the nursing supervisor.

B) tell the patients that the nurse is going through some difficult times.

C) tell the nurse the comments the patients have been making about her.

D) ask each of the patients to talk more about the nurse's behavior.

Correct Answer is: C

Explanation : The charge nurse should discuss patient comments with the nurse and work with
the nurse to develop a plan that promotes change in the behavior.

130) Which of the following responses of a female patient who is codependent and has low self-
esteem indicates that nursing interventions have been successful?

Options:

A) The patient refuses help from her child with meal preparation.

B) The patient regularly prepares refreshments for her reading club.

C) The patient encourages her 16-year-old daugther to prepare her own breakfast.

D) The patient seeks other family members' approval prior to preparing meals.

Correct Answer is: B


Explanation : The nurse should instruct the parents that correct positioning to prevent sudden
infant death syndrome (SIDS) is the supine position or the side-lying position.

131) Which of the following activities, if performed by the nurse, is an example of primary
preventions?

Options:

A) Promoting independence in the elderly

B) Assessment of family growth and development

C) Identification of problematic behavior in children

D) Screening for depression

Correct Answer is: C

Explanation : Identification of problematic behavior in children is a primary prevetion.As a nurses


identifies through the history of the child from the parent will allow her to plan for preventing
the further problems.

132) A Certified Nursing Assistant (CNA) who is bathing a patient with end-stage cirrhosis notices
that the patient has become disoriented and reports this observation to the nursing supervisor.
Recognizing this as a change from baseline, the supervisor would initially

Options:

A) limit the patient's physical activity.

B) perform a mental status examination.

C) document the change in orientation in the medical record.

D) Notify the physician.

Correct Answer is: A

Explanation : Manifestations of viral hepatitis include lethargy, irritability, drowsiness and


anemia. One of the primary nursing diagnoses for this patient would be activity intolerance
related to fatigue. The patient should be instructed to maintain adequate rest to conserve
energy.

133) A patient who is admitted to the hospital gives the nurse an advance directive. The nurse
should understand that an advance directive is

Options:

A) a written statement by the patient that defines acceptable care if the patient becomes
incapacitated.

B) a statement identifying the patient as an organ donor.

C) the name of the person designated by the patient to make health-related decisions should the
patient become incapacitated.

D) a written statement authorizing a particular surgical procedure.

Correct Answer is: A

Explanation : An advance directive3 is a written document that contains directives of the


person's choices regarding end of life care. A person must have the cognitive and communicative
abilities to execute decisions regarding their desires. It includes wishes for treatment options
should the person become unable to do so.

134) The registered nurse's signature as a witness on an informed consent indicates that the
patient

Options:

A) was medicated for pain before the consent was signed.

B) has been informed regarding the procedure.

C) voluntarily agreed to having the procedure performed.

D) can described how the surgical procedure will be done.

Correct Answer is: B

Explanation : In order for an informed consent to be valid, three basic criteria must be met. The
patient's decision must be voluntary, the patient must be informed, and the patient must be
competent to understand the information and alternatives. The registered nurse's signature as a
witness indicates these criteria were met.

135) A psychiatric patient continues to disrupt the unit milieu by pacing up and down the hall.
The nurse responds by placing the patient in the seclusion room. As a result of her actions, the
nurse may be held responsible for which of the following legal implications?

Options:

A) False imprisonment

B) Invasion or privacy

C) Battery

D) Defamation of character

Correct Answer is: A

Explanation : There is no indication for the use of seclusion with this patient. The use of
seclusion or restrain that is not defensible as being necessary and in the client's best interest
may result in false imprisonment of the client and liability for the nurse.

136) Which of the following measures is most appropriate for a nurse to take a prevent injury in
a patient who is confused?

Options:

A) change the patient's environment

B) Keep the bed in the lowest position

C) Administer lorazepam (Ativan) as ordered

D) Apply a soft restraint on the patient's wrist

Correct Answer is: B

Explanation : Placing the bed in the low position is an important step in a falls prevention
program. The bed should be as low to the floor as possible to decrease falls and prevent injury.
137) A nurse is assigned to all of the following patients. Which patient should the nurse assess
first?

Options:

A) The patient who has an intravenous medication due in 30 minutes

B) The patient requesting medication for chest pain

C) The patient who is scheduled to go to surgery within the hour

D) The patient who has a temperature of 101 degrees F

Correct Answer is: B

Explanation : Chest pain could be a sign of a myocardial infarction or life-threatening pulmonary


embolus. The nurse should assess the patient for changes in the blood pressure, heart rate,
rhythm, and electrocardiogram (EKG). Assessment of accompanying symptoms and precipitating
factors to the chest pain should be performed.

139) Which of the following actions would be essential for the nurse to take when caring for a
patient in restraints in the psychiatric unit?

Options:

A) Document the events leading to the use of restraints

B) Obtain a physician's order within 24 hours of the restraints being applied

C) Check the patient every four hours while restrained

D) Replace the restraints every 24 hours to ensure proper fit

Correct Answer is: A

Explanation : It is critical to document the events leading up to the use of restraints. Specifically,
what was happening immediately before the event and what other measures were attempted
before restraints were used. Clear danger to self or others must be evident in the
documentation.
141) Which of the following explanations would the nurse give to a patient regarding the role of
the case manager?

Options:

A) The case manager negotiates insurance benefits with the hospital.

B) The case manager coordinates both inpatient hospitalization and home care.

C) The case manager makes daily patient assignments for staff.

D) The case manager decides what treatments are essential.

Correct Answer is: B

Explanation : Case managers plan, coordinate and monitor services to meet the needs of the
client. Case managers pull together available resources to provide the client with the best help.
Case management is based on a compilation of services that contains costs and improves the
quality of care while reducing fragmentation and duplication of services.

142) The nurse should recognize that a patient who is unable to remember being raped by her
brother when she was 10 years old is using which of the following ego defense mechanisms?

Options:

A) Undoing

B) Regression

C) Repression

D) Compensation

Correct Answer is: C

Explanation : Repression is the exclusion of unpleasant or unwanted experiences, emotions, or


ideas from conscious awareness. It is the first psychological defense against anxiety. The client's
inability to remember the details of the rape protects her from emotional pain.

143) Which of the following behaviors by a nurse who is suspected of being impaired would
support a nursing diagnosis of ineffective individual coping?
Options:

A) Frequent illnesses

B) Decreased job performance

C) Increased food consumption

D) Verbal manipulation

Correct Answer is: B

Explanation : Impairment due to substance use results in an inability to function in various roles,
including the job. This would result in decreased performance.

145) A client has been admitted to the mental health unit on a voluntary basis. The client has
reported a history of depression over the past 5 years. Which of the following questions by the
nurse would elicit the most thorough assessment data regarding the client’s recent sleeping
patterns?

Options:

A) “Have you been having trouble sleeping at home?”

B) “Did you sleep last night?”

C) “Tell me about your sleeping patterns.”

D) “You look as if you could use some sleep.”

Correct Answer is: C

Explanation : Option 3 is an open-ended question and provides the client the opportunity to
express thoughts and feelings. Options 1 and 2 could lead to a one-word answer that would not
provide thorough assessment data. Additionally, one night of sleep may not tell the nurse how
the pattern has been over time. Anyone may or may not sleep well for one night and that sleep
or loss of sleep does not indicate a problem. Option 4 could be interpreted by the depressed
person as a negative statement and could block communication needed for a thorough
assessment.
146) A nurse is planning the discharge instructions from the emergency department for an adult
client who is a victim of family violence. The nurse understands that the discharge plans must
include:

Options:

A) Instructions to call the police the next time the abuse occurs

B) Exploration of the pros and cons of remaining with the abusive family member

C) Specific information regarding “safe havens” or shelters in the client’s neighborhood

D) Specific information about self defense classes.

Correct Answer is: C

Explanation : Any of the options might be included in the discharge plan at some point if long-
term therapy or a long-term relationship with the nurse is established. The question refers to an
emergency department setting. It is most important to assist victims of abuse with identifying a
plan for how to remove self from harmful situations should they arise again. An abused person is
usually reluctant to call the police. Teaching the victim to fight back (as in the use of self-
defense) is not the best action when dealing with a violent person.

147) A nurse is assisting in providing a form of psychotherapy in which the client acts out
situations that are of emotional significance. The nurse understands that this form of therapy is
known as:

Options:

A) Reality therapy

B) Short-term dynamic psychotherapy

C) Psychoanalytic therapy

D) Psychodrama

Correct Answer is: D

Explanation : Psychodrama involves enactment of emotionally charged situations. Reality


therapy is used for individuals with cognitive impairment. Both short-term dynamic
psychotherapy and psychoanalytic therapy depend on techniques drawn from psychoanalysis.
148) A hospitalized client has participated in substance abuse therapy group sessions. The nurse
is monitoring the client’s response to the substance abuse sessions. Which statement by the
client would best indicate that the client has assimilated session topics, coping response styles,
and has processed information effectively for self-use?

Options:

A) “I know I’m ready to be discharged; I feel like I can say no and leave a group of friends if they
are drinking…. No problem.”

B) “This group has really helped a lot. I know it will be different when I go home. But I’m sure
that my family and friends will all help me, like the people in this group have. They’ll all help
me….I know they will……They won’t let me go back to old way

C) “I’m looking forward to leaving here; I know that I will miss all of you. So, I’m happy and I’m
sad. I’m excited and I’m scared. I know that I have to work hard to be strong and that everyone
isn’t going to be as helpful as you people. I know it isn’

D) “I’ll keep all my appointments; I’ll do everything I’m supposed to… Nothing will go wrong that
way.”

Correct Answer is: C

Explanation : In the defense mechanism of denial, the person denies reality. There can be
varying degrees of this denial. In option 3, the client is expressing real concern and ambivalence
about discharge from the hospital. The client is real in the appraisal about the changes the client
will have to initiate in lifestyle as well as the fact that the client has to work hard and develop
new friends and meeting places. Option 1 identifies denial. In option 2, the client is relying
heavily on others, and the client’s locus of control is external. In option 4, the client is concrete
and procedure oriented; again, the client verbalizes denial.

149) An older client who has never been hospitalized before is to have a 12-lead
electrocardiogram. (ECG). The nurse would alleviate the client’s anxiety about the test by giving
which of the following explanations?

Options:

A) “The ECG can give the doctor information about what might be wrong with your heart.”

B) “It’s important to lie still during the procedure.”


C) “It should only take about 20 minutes to complete the ECG tracing.”

D) “The ECG electrodes are painless and will record the electrical activity of the heart.”

Correct Answer is: D

Explanation : The ECG uses painless electrodes, which are applied to the chest and limbs. It takes
less than 5 minutes to complete and requires the client to lie still. The ECG measures the heart’s
electrical activity to determine rate, rhythm, and a variety of abnormalities. Options 1 and 2 are
factual statements but are not stated to reduce anxiety.

150) A visitor who is visiting her husband on your nursing unit asks the nurse about another
patient on the unit, who happens to be her friend. The visitor states that she saw this patient’s
name on the computer screen that another nurse was using at the desk. What should you do?

Options:

A) Tell the visitor that the friend is a patient on the unit but do not disclose any further
information.

B) Discuss the matter with the other nurse, reminding him or her not to leave client information
in view of visitors.

C) Tell the visitor that she should not read information that is confidential and then notify
security.

D) Ask the friend to come to the client’s room to meet with the wife after your are finished
administering medications

Correct Answer is: B

Explanation : Leaving personal information in view of another person is a breach of


confidentiality. The nurse should approach the nurse at the computer and inform him or her of
the incident.

151) Monica, a nurse educator, is speaking to a group of nursing students about effective
communication techniques. Which of the following would Monica state is the goal of therapeutic
communication?

Options:
A) Obtaining information, developing trust, and shoeing caring.

B) Giving advice, data collecting, and developing a communication style.

C) Self-disclosure, sympathy, and obtaining information.

D) Validation, sympathy, and developing trust.

Correct Answer is: A

Explanation : Therapeutic communication is client focused; goal directed; and includes an


appropriate use of self, which includes empathy versus sympathy. Therapeutic communication
conveys caring without crossing the boundaries of communication techniques unique to social
and personal relationships.

152) Margaret, the nursing team leader, overhears comments made between two nurses. Gilles,
an RN, repeatedly make remarks that are focused on Stephen’s skin color and race. Stephen is
observably offended. Which of the following actions by Margaret would demonstrate an
understanding of promoting a quality practice environment?

Options:

A) Speak to Gilles directly, pointing out that he harassing Stephen and that it will not be
tolerated.

B) In Gilles’ mailbox, leave a pamphlet that addresses how to deal with harassment and
discrimination.

C) Seek out some posters for the unit that reflect racial diversity and post them at a time when
Stephen is not working.

D) Nothing; Stephen must submit a formal complaint to the human rights department before
anything will be done.

Correct Answer is: A

Explanation : It is the nursing manager’s responsibility to intervene and advise Gilles that his
comments are harassing and inappropriate and will not be tolerated in the work environment.
This discussion should be clearly documented and the situation closely monitored in case Gilles
makes similar comments in the future.
153) An RN on a surgical unit learns that a group of nursing students will be coming on the unit
for 7 weeks with an instructor. The nurse tells the manager that she “wants nothing to do with
those students” and tells the manager not to assign a student to her patients. Which of the
following is the most appropriate response by the manager to this request?

Options:

A) “I understand how time consuming it can be to have a student. I will ask someone else.”

B) “The choice is yours. You are under no obligation to work with nursing students.”

C) “Most nurses like working with students. I thought you would like to as well.”

D) “As a nurse, you have a professional obligation to share knowledge with students.”

Correct Answer is: D

Explanation : Of the choices provided, D is the most appropriate. All nurses have a responsibility
to provide teaching and learning opportunities to students. The nurse manager should further
explore with the nurse the reasons why she does not want to work with a student. Strategies
should be designed to support both the nurses and the students on this unit without resulting in
any negative outcomes for clients.

154) A 4-year-old child who was recently hospitalized is brought to the clinic by his mother for a
follow-up visit. The mother tells the nurse that the child has begun to wet the bed ever since the
child was brought home from the hospital. The mother is concerned and asks the nurse what to
do. The appropriate nursing response is which of the following?

Options:

A) “You need to discipline the child.”

B) “This is a normal occurrence following hospitalization

C) “We will need to discuss this behavior with the physician.”

D) “The child probably has developed a urinary tract infection.”

Correct Answer is: B

Explanation : Regression can occur in a preschooler and is most often a result of the stress of the
hospitalization. Its is best to accept the regression if it occurs. Parents may be overly concerned
about the regressive behavior and should be told that regression is normal following
hospitalization. It is premature to discuss the situation with the physician. Options 1 and 4 are
iappropriate responses to the mother.

155) A young adult male client with a spinal cord injury tells the nurse, “It’s so depressing that I’ll
never get to have sex again.” The nurse replies in a realistic way by making which of the following
statements to the client?

Options:

A) “You’re young, so you’ll adapt to this more easily than if you were older.”

B) “It must feel horrible to know you can never have sex again.”

C) “It is still possible to have a sexual relationship, but it is different.”

D) “Because of body reflexes, sexual functioning will be no different than before.”

Correct Answer is: C

Explanation : It is possible to have a sexual relationship after a spinal cord injury, but it is
different than what the client experienced before the injury. Males may experience reflex
erections, although they may not ejaculate. Females can have adductor spasm. Sexual
counseling may help the client adapt to changes in sexuality after spinal cord injury.

156) A client recovering from a cerebrovascular accident (CVA) has become irritable and angry
regarding limitations. Which of the following is the best nursing approach to help the client
regain motivation to succeed?

Options:

A) Allow longer and more frequent visitation by the spouse

B) Use supportive statements to correct the client’s behavior

C) Tell the client that the nurse are experienced and know how the client feels

D) Ignore the behavior, knowing that the client is grieving

Correct Answer is: B


Explanation : Clients who have experienced a CVA have many and varied needs. The client may
need his or her behavior pointed out so that correction can take place. It is also important to
support and praise the client for accomplishments. Spouses of a CVA client are often grieving;
therefore, more visitation may not be helpful. Additionally, short visits are often encouraged.
Stating that the nurse knows how the client feels is inappropriate. The client’s behavior should
not be ignored.

157) A client with myasthenia gravis is ready to return home. The client confides that she is
concerned that her husband will no longer find her physically attractive. The nurse would include
in the plan of care to:

Options:

A) Encourage the client to start a support group

B) Insist that the client reach out and face this fear

C) Tell the client not to dwell on the negative

D) Encourage the client to share her feelings with her husband

Correct Answer is: D

Explanation : Sharing feelings with her husband directly addresses the issue of the question.
Encouraging the client to start a support group will not address the client’s immediate and
individual concerns. Options 2 and 3 are blocks to communication and avoid the client’s concern.

158) A client who has a spinal cord injury and is paralyzed from the neck down frequently makes
lewd sexual suggestions and uses profanity. The nurse interprets that the client is inapproprietly
using the defense mechanism of displacement and determines that the appropriate nursing
diagnoses for this client is:

Options:

A) Ineffective coping

B) Risk for Disuse Syndrome

C) Impaired Environmental Interpretation Syndrome

D) Disturbed Body Image


Correct Answer is: A

Explanation : The definition of Ineffective Coping is the “State in which an individual


demonstrates impaired adaptive behaviors and problem-solving abilities in meeting life’s
demands and roles. “By displacing feelings onto the environment instead of in a constructive
fashion, this nursing diagnosis clearly applies in this situation. Options 2 and 3 have no relation
to this situation. Option 4 may be appropriate, but it has nothing to do with the displacement
that the client is currently using.

159) A client with a T-1 spinal cord injury has just learned that the cord was completely severed.
The client says, “I’m no good to anyone. I might as well be dead.” The nurse makes which
appropriate response to the client?

Options:

A) “It makes me uncomfortable when you talk this way.”

B) “I’ll ask the psychologist to see you about this.”

C) “You’re not a useless person at all.”

D) “You are feeling pretty bad about things right now.”

Correct Answer is: D

Explanation : Restating and reflecting keeps the lines of communication open and encourages
the client to expand on current feelings of unworthiness and loss that require exploration. The
nurse can block communication by showing discomfort, disapproval, or by postponing discussion
of issues. Grief is a common reaction to loss of function. The nurse facilities grieving through
open communication.

160) A client with a recent complete T-4 spinal cord transaction tells the nurse that he will walk
again as soon as the spinal shock resolves. Which of the following will provide the most accurate
basis for planning a response to the client?

Options:

A) In order to speed acceptance, the client needs reinforcement that he will not walk again.

B) The client needs to move through the grieving process rapidly in order to benefit from
rehabilitation

C) The client is projecting by insisting that walking is the rehabilitation goal.

D) Denial can be protective while the client deals with the anxiety created by the new disability.

Correct Answer is: D

Explanation : In the adjustment period during the first few weeks after spinal cord injury, clients
may use denial as a defense mechanism. Denial may decrease anxiety temporarily and is a
normal part of grieving. After the spinal shock resolves, prolonged or excessive use of denial may
impair rehabilitation. However, rehabilitation programs include psychological counseling to deal
with denial and grief.

161) A client who has had a spinal fusion with insertion of hardware is extremely concerned with
the perceived lengthy rehabilitation period. The client expresses concerns about finances and
the ability to return to prior employment. The nurse understands that the client’s needs could
best be addressed by referral to the:

Options:

A) Surgeon

B) Clinical nurse specialist

C) Social worker

D) Physical therapist

Correct Answer is: C

Explanation : Following spinal surgery, concerns about finances and individual is able to provide
information about resources available to the client. The physical therapist has the best
knowledge of techniques for increasing mobility and endurance. The clinical nurse specialist and
surgeon would not have the necessary information related to financial resources.

162) A nurse is caring for a client who is recovering from the signs and symptoms of autonomic
dysreflexia (hyperreflexia). The nurse makes which thrapeuticstatement to the client?

Options:
A) “I’m sure you now understand the importance of preventing this from occurring.”

B) “Now that this problem is taken care of, I’m sure you’ll be fine.”

C) “How could your home care nurse let this happen?”

D) “I have some time if you would like to talk about what happened to you.”

Correct Answer is: D

Explanation : Option 4 encourages the client to discuss feelings. Option 1 and 3 shows
disapproval and option 2 provides false reassurance. These are nontherapeutic techniques.

163) While assisting a spinal cord injury client with activities of daily living, the client states, “I
can’t do this; I wish I were dead.” The nurse makes which therapeutic response to the client?

Options:

A) “Let’s wash your back now.”

B) “You wish you were dead?”

C) I’m sure you are frustrated, but things will work out just fine for you.”

D) “Why do you day that?”

Correct Answer is: B

Explanation : Clarifying is a therapeutic technique involving restating what was said to obtain
addition information. Option 1 changes the subject. In option 3, false reassurance is offered. By
asking “why” (option 4), the nurse puts the client on the defensive. Options 1,3, and 4 are
nontherapeutic and block communication.

164) A client with a spinal cord injury makes the following comments. Which comment warrants
additional intervention by the nurse?

Options:

A) “I’m so angry this happened to me.”

B) “I know I will have to make major adjustments in my life.”


C) “I would like my family members to be here for my teaching sessions.”

D) “I’m really looking forward to going home.”

Correct Answer is: A

Explanation : It is important to allow the client with a spinal cord injury to verbalize or her
feelings. If the client indicates a desire to discuss feelings, the nurse should respond
therapeutically. Options 2 and 3 indicate that the client understands changes that will be
occurring and that family involvement is best. No data in the question indicates that the client
will not be going home, therefore this comment does not require further intervention.

165) A nurse is caring for a client with a grade II (mild) cerebral aneurysm rupture. The client
becomes restless and anxious before visiting hours. The nurse determines that the client’s
behavior is likely related to:

Options:

A) The severity of the aneurysm rupture

B) Disabled family coping

C) Disturbed body image

D) Spiritual distress

Correct Answer is: C

Explanation : A grade II cerebral aneurysm rupture is a mild bleed in which the client remains
alert but has nuchal rigidity with possible neurological deficits, depending on the area of the
bleed. Because these clients remain alert, they are acutely aware of the neurological deficits and
frequently have some degree of body image disturbance. No data in the question indicates that
the client’s behavior is related to options 1, 2, or 4.

166) A nurse is assessing the level of consciousness in a child with a head injury and documents
that the child is obtunded. Based on this documentation, which observation did the nurse note?

Options:

A) The child is unable to recognize place or person


B) The child is unable to think clearly and rapidly

C) The child requires considerable stimulation for arousal

D) The child sleeps unless aroused and once aroused has limited interaction with the
environment.

Correct Answer is: D

Explanation : If the child is obtunded, the child sleeps unless aroused and once aroused has
limited interaction with the environment. Option 1 describes disorientation. Option 2 describes
confusion. Option 3 describes stupor.

167) A nurse is obtaining a history on a client admitted to the hospital with a thrombotic
cerebrovascular accident (CVA). The nurse assesses the client, knowing that before the CVA
occurred, the client most likely experienced:

Options:

A) Transient hemiplegia and loss of speech

B) Throbbing headaches

C) Unexplained episodes of loss of consciousness

D) No symptoms at all

Correct Answer is: A

Explanation : Cerebral thrombosis does not occur suddenly. In the few hours or days preceding a
thrombotic VCA, the client may experience a transient loss of speech, hemoplegia, or
paresthesias on one side of the body. Other signs and symptoms of thrombotic CVA vary, but
may include idzziness, cognitive changes or seizures. Headache is rare, and loss of consciousness
is not likely to occur.

168) A client with thrombotic cerebrovascular accident (CVA) experiences periods of emotional
lability. The client alternately laughs and cries, and intermittently becomes irritable and
demanding. The nurse interprets that this behavior indicates:

Options:
A) That the problem is likely to get worse before it gets better

B) That the clients is experiencing the usual sequelae of a CVA

C) The client is not adapting well to the disability

D) The client is experiencing side effects of prescribed anticoagulants

Correct Answer is: B

Explanation : Following CVA, the client often experiences periods of emotional liability, which is
characterized by sudden bouts of laughing or crying, or by irritability, depression, confusion, or
being demanding. This is a normal part of the clinical picture for the client with this health
problem, although it may be difficult for health care personnel and family members to deal with.
The other options are incorrect.

169) A nurse is caring for a client with myasthenia gravis. The client is vomiting and complaining
of abdominal cramps and diarrhea. The nurse also notes that the client is hypotensive and is
experiencing facial muscle twitching. The nurse interprets that these symptoms are compatible
with:

Options:

A) Systemic infection

B) A reaction to plasmapheresis

C) Cholinergic crisis

D) Myasthenic crisis

Correct Answer is: C

Explanation : Signs and symptoms of cholinergic crisis include nausea, vomiting, abdominal
cramping, diarrhea, blurred vision, pallor, facial muscle twitching, papillary myosis, and
hypotension. It is caused by overmedication with cholinergic (anticholinesterase) medications
and is treated by withholding medications. Myasthenic crisis is an exacerbation of myasthenic
symptoms caused by undermedication with anticholinesterase medications. There is no data in
the question to support options 1, 2, and 2.
170) A nursing student is preparing to conduct a clinical conference regarding cerebral palsy.
Which characteristic related to this disorder will the student plan to include in the discussion?

Options:

A) Cerebral palsy is a chronic disability characterized by a difficulty in controlling muscles

B) Cerebral palsy is an infectious disease of the central nervous system

C) Cerebral palsy is an inflammation of the brain as a result

D) Normal verbal but abnormal nonverbal communication

Correct Answer is: A

Explanation : Cerebral palsy is a chronic disability characterized by a difficulty in controlling


muscles because of a abnormality in the extapyramidal or pyramidal motor system. Meningitis is
an infectious process of the central nervous system. Encephalitis is an inflammation of the brain
that occurs as a result of viral illness or central nervous system infections. Down syndrome is an
example of a congenital condition that results in moderated to severe retardation.

171) A client with a cerebral vascular accident (CVA) is prepared for discharge from the hospital.
The physician has prescribed range-of-motion (ROM) exercises for the client’s right side, In
planning for the client’s care, the home care nurse:

Options:

A) Considers the use of active, passive, or active-assisted exercises in the home.

B) Implements ROM exercises to the point of pain for the client.

C) Encourages the client to be dependent on the home care nurse to complete the exercise
program

D) Develops a schedule of ROM exercises every 2 hours while awake even if the client is fatigued.

Correct Answer is: A

Explanation : The home care nurse must consider all forms of ROM for the client. Even if the
client has right hemiplegia, the client can assist in some of his or her own rehabilitative care. In
addition, the goal in home care nursing is for the client to assume as much self-care and
independence as possible. The nurse needs to teach so that the client becomes of self-reliant.
Options 2 and 4 are incorrect from a physiological standpoint.

172) A client with a cerebrovascular accident (CVA) has episodes of coughing while swallowing
liquids. The client has developed a temperature of 101oF, oxygen saturation of 91% (down from
98% previously), slight confusion, and noticeable dyspnea. The nurse would take which most
appropriate action?

Options:

A) Administer a bronchodilator ordered on a prn basis

B) Administer an acetaminophen (Tylenol) suppository

C) Encourage the client to cough and deep breathe

D) Notify the physician.

Correct Answer is: D

Explanation : The client is exhibiting clinical signs and symptoms of aspiration, which include
fever, dyspnea, decreased arterial oxygen levels, and confusion. Other symptoms that occur with
this complication are difficulty in managing own saliva, or coughing or choking while eating.
Because the client has developed a complication requiring medical intervention, the most
appropriate action is to contact the physician

173) A nurse is caring for a client who begins to experience seizure activity while in bed. The
nurse implements which action to prevent aspiration?

Options:

A) Loosens restrictive clothing

B) Removes the pillow and raises the padded side rails

C) Raises the head of the bed

D) Positions the client on the side if possible, with the head flexed forward

Correct Answer is: D

Explanation : Positioning the client on one side with the head flexed forward allows the tongue
to fall forward and facilitates drainage of secretions, which could help prevent aspiration. The
nurse would also remove restrictive clothing and the pillow, and raise the padded side rails, if
present, but these actions would not decrease the risk of aspiration. Rather, they are general
safety measures to use during seizure activity. The nurse would not raise the client’s head of
bed.

174) The family or a client with Parkinson’s disease tells the nurse that the client is having
difficulty adjusting to the disorder and that they do not know what to do to help. The nurse
advises the family that which of the following would be therapeutic in assisting the client to cope
with the disease?

Options:

A) Encourage and praise client efforts to exercise and perform activities of daily living (ADLs)

B) Cluster activities at the end of the day when the client is restless and bored

C) Plan only a few activities for the client during the day

D) Assist the client with ADLs as much as possible

Correct Answer is: A

Explanation : The client with Parkinson’s disease has a tendency to become withdrawn and
depressed, which can be limited by encouraging the client to be an active participant in his or
her own care. The family should also give the client encouragement and praise for perseverance
in these efforts. The family should plan activities intermittently throughout the day to inhibit
daytime sleeping and boredom.

175) A nurse is performing an assessment on a client being admitted to the hospital. The client
has right-sided weakness, aphasia, and urinary incontinence. One of the client’s family members
states, “If this is a stroke, it’s the kiss of death.” The nurse makes which response to the family
member?

Options:

A) “Wait until the doctor gets here to think like that.”

B) “A stroke is not the kiss of death.”

C) “You feel as if your parent is dying?”


D) “These symptoms may be reversible.”

Correct Answer is: C

Explanation : Option 3 allows the family member to verbalize and begin to cope and adapt to
what is happening. By restating, the nurse is able to clarify the family member’s feelings and
begin to offer information that will help ease some of the fears that he or she may face at the
moment. Options 1 and 2 offer disapproval and put the family member’s feeling on hold. Option
4 provides false hope at this point.

176) A client with myasthenia gravis is having difficulty with the motor aspects of speech. The
client has difficulty forming words, and the voice has a nasal tone. The nurse would use which
communication strategy when working with this client?

Options:

A) Repeat what the client said to verify the message

B) Encourage the client to speak quickly

C) Nod continuously while the client is speaking

D) Engage the client in lengthy discussions to strengthen the voice

Correct Answer is: A

Explanation : The client has speech that is nasal in tone and dysarthritic because of cranial nerve
involvement of the muscles governing speech. The nurse listens attentively and verbally verifies
what the client has said. Other helpful techniques are to ask questions requiring a yes or no
response, and to develop alternative communication methods (eg., letter board, picture board,
pen and paper, flash cads). Encouraging the client to speak quickly is inappropriate and
counterproductive. Continuous nodding may be distracting and is unnecessary. Lengthy
discussion will tire the client rather than strengthen the voice.

178) A client with myasthenia gravis is having difficulty with the motor aspects of speech. The
client has difficulty forming words, and the voice has a nasal tone. The nurse would use which
communication strategy when working with this client?

Options:
A) Repeat what the client said to verify the message

B) Encourage the client to speak quickly

C) Nod continuously while the client is speaking

D) Engage the client in lengthy discussions to strengthen the voice

Correct Answer is: A

Explanation : The client has speech that is nasal in tone and dysarthritic because of cranial nerve
involvement of the muscles governing speech. The nurse listens attentively and verbally verifies
what the client has said. Other helpful techniques are to ask questions requiring a yes or no
response, and to develop alternative communication methods (eg., letter board, picture board,
pen and paper, flash cads). Encouraging the client to speak quickly is inappropriate and
counterproductive. Continuous nodding may be distracting and is unnecessary. Lengthy
discussion will tire the client rather than strengthen the voice.

179) A home care nurse visits an order client who has acute gouty arthritis. Indomethacin
(Indocin) has been prescribed for the client, and the nurse teaches the client about the
medication. Which statement by the client indicates the further teaching is necessary?

Options:

A) “I can take a pill whenever I need to for pain.”

B) “I need to call the doctor if I notice a rash.”

C) “I’ll rest if I am having pain.”

D) “I’ll watch for any swollen feet or fingers or any stomach distress.”

Correct Answer is: A

Explanation : Indomethacin (Indocin) may be prescribed to treat gouty arthritis. It may alleviate
pain but is administered on a scheduled time frame, not a prn schedule. When pain occurs, the
client will usually limit movement and rest. A rash should be reported because it could indicate
hypersensitivity to the medication. The client should be instructed to monitor for swelling and
gastric distress, which can be caused by the medication.
180) A client with a head injury and a feeding tube continuously tries to remove the tube. The
nurse contacts the physician who prescribes the use of restraints. After checking the agency’s
policy and procedure regarding the use of restraints, the nurse uses which method in restraining
the client?

Options:

A) Mitten restraints

B) Wrist restraints

C) Waist restraints

D) Vest restraint

Correct Answer is: A

Explanation : Mitten restraints are useful for this client because the client cannot pull against
them, creating resistance that could lead to increased intracranial pressure (ICP). Wrist restraints
cause resistance. Vest and waist restraints prevent the client from getting up or falling out of bed
but do nothing to limit hand movement.

181) A client has an order for seizure precautions, and a nursing student develops a plan of care
for the client. The registered nurse reviews the plan of care with the student and identifies which
incorrect intervention?

Options:

A) Monitor the client closely while the client is showering

B) Push the lock-out button on the electric bed to keep the bed in lowest position

C) Keep all the light on in the room at night

D) Assist the client to ambulate in the hallway

Correct Answer is: C

Explanation : A quiet, restful environment is provided as part of seizure precautions. This


includes undisturbed times for sleep, while using a nightlight for safety. The client should be
accompanied during activities such as bathing and walking, so that assistance readily available
and injury is minimized if a seizure begins. The bed is maintained in low position for safety.
182) Auranofin (Ridaura) is prescribed for a client with rheumatoid arthritis, and the nurse
monitors the client for signs of an adverse effect related to the medication. Which of the
following indicates an adverse effect?

Options:

A) Nausea

B) Diarrhea.

C) Anorexia

D) Proteinuria

Correct Answer is: D

Explanation : Auranofin (Ridaura) is a gold preparation that is used as an antirheumatic. Gold


toxicity is an adverse effect and is evidenced by decreased hemoglobin, leucopenia, reduced
granulocyte counts proteinuria, hematuria, stomatitis, glomerulonephritis, nephritic syndrome,
or cholestatic jaundice. Anorexia, nausea, and diarrhea are frequent side effects of the
medication.

183) A nurse is caring for a client with an acute head injury. The nurse carefully assesses which
neurological sign as the most sensitive indicator of neurological status?

Options:

A) Vital signs

B) Level of consciousness

C) Sensory function

D) Motor function

Correct Answer is: B

Explanation : The level of consciousness is the most sensitive indicator of neurological status. An
alteration in the level of consciousness occurs before any other changes in neurologic signs or
vital signs. Vital sign changes occur late.
184) A nurse is caring for the client with silicosis who has massive pulmonary fibrosis. The nurse
monitors the client for emotional reactions related to the chronic respiratory disease. Which
emotional reaction if expressed by the client would indicate a need for immediate intervention?

Options:

A) Anxiety

B) Ineffective coping

C) Depression

D) Suicidal ideation

Correct Answer is: A

Explanation : Common emotional reactions to a disease such as massive pulmonary fibrosis may
be the same as for chronic air flow limitation and include anxiety, ineffective coping, and
depression. Suicidal ideation is not a normal emotional reaction with this condition. If it is
expressed, it warrants immediate intervention.

185) A client with Bell’s palsy is distressed about the change in facial appearance. The nurse tells
the client about which characteristic of Bell’s palsy to help the client cope with the disorder?

Options:

A) The symptoms will completely go away once the tumor is removed.

B) It usually resolves when treated with vasodilator medications.

C) It is similar to strike, but all symptoms will go away eventually

D) It is not caused by stroke, and many clients recover in 3 to 5 weeks

Correct Answer is: D

Explanation : Clients with Bell’s palsy should be reassured that they have not experienced a
stroke and that symptoms often disappear spontaneously in 3 to 5 weeks. The client is given
supportive treatment for symptoms. It is not usually caused by a tumor, and the treatment does
not involve administering vasodilators.
186) A nurse is monitoring a client with multiple sclerosis who is receiving baclofen (Lioresal).
Which assessment finding would indicate a therapeutic response from the medication?

Options:

A) Increased muscle tone and strength

B) Decreased nausea

C) Decreased muscle spasms

D) Increased range of motion of all extremities

Correct Answer is: C

Explanation : Baclofen is a skeletal muscle relaxant and acts at the spinal cord level to decrease
the frequency and amplitude of muscle spasms in clients with spinal cord injuries or disease, or
multiple sclerosis. Options 1, 2, and 4 are unrelated to the effects of this medication.

187) A nurse is in the room with a client when a seizure begins. The client’s entire body becomes
rigid, and the muscles in all four extremities alternate between relaxation and contraction.
Following the seizure, the nurse documents that the client has experienced a(n):

Options:

A) Absence seizure

B) Tonic-clonic seizure

C) Partial seizure

D) Complex partial seizure

Correct Answer is: B

Explanation : Tonic-clonic seizures are characterized by body rigidity (tonic phase) followed by
rhythmic jerky contraction and relaxation of all body muscles, especially hose of the extremities
(clonic phase). There are two types of complex partial seizures complex partial seizures with
automatisms and partial seizures evolving into generalized seizures. Complex partial seizures
with automatisms include purposeless repetitive activities such as lip smacking, chewing, or
patting the body. Partial seizures evolving into a generalized seizure begin locally and then
spread through the body. Absence seizures are characterized by a sudden lapse of consciousness
for approximately 2 to 10 seconds and a blank facial expression.

188) A nurse is reviewing the nursing care plan for a client with a right cerebrovascular accident
(CVA) who has left-sided deficits. The nurse notes a nursing diagnosis of Unilateral Neglect. The
nurse would tell a family member who is assisting the client that it would be least helpful to do
which of the following?

Options:

A) Approach the client from the right side

B) Teach the client to scan the environment

C) Move the commode and chair to the left side

D) Place bedside articles on the left side

Correct Answer is: A

Explanation : Unilateral neglect is an unawareness of the paralyzed side of the body, which
increases the client’s risk for injury. The nurse’s role is to refocus the client’s attention to the
affected side. Personal care items, belongings, a bedside chair, and a commode are all placed on
the affected side. The client is taught to scan the environment to become aware of that half of
the body and is approached on that side by family and caregivers as well.

189) A nurse is evaluating the status of client with myasthenia gravis. The nurse interprets that
the client’s medication regime may not be optimal if the client continues to experience fatigue
that occurs:

Options:

A) Before meals and at the end of the day

B) Early in the morning and late in the day

C) Following exertion and at the end of the day

D) Early in the morning and before lunch


Correct Answer is: C

Explanation : The client with myasthenia gravis has weakness after periods of exertion and near
the end of the day. Medication therapy should assist in alleviating the weakness. The medication
regime may not be optimal if the client continues to experience fatigue. The nurse also works
with the client to space out activities to conserve energy and regain muscle strength by resting
between activities. The client is also instructed to take medication as prescribed.

190) A client with a ruptured intracranial aneurysm has surgery delayed and is still maintained
on bed rest with subarachnoid precautions in place. The nurse should question an order for
which of the following medications if prescribed for this client?

Options:

A) Aminocaproic acid (Amicar)

B) Heparin sodium (Heparin)

C) Nimodipine (Nimotop)

D) Docusate sodium (Colace)

Correct Answer is: B

Explanation : The nurse should question an order for heparin sodium, which is an anticoagulant.
This medication could place the client at risk for rebleeding. Aminocaproic acid is an
antifibrinolytic agent that prevents clot breakdown or dissolution. It may be prescribed after
ruptured intracranial aneurysm and subarachnoid hemorrhage if surgery is delayed or
contraindicated. Nimodipine is a calcium-channel is delayed or contraindicated. Nimodipine is a
calcium-channel blocking agent that is useful in the management of vasospasm associated with
cerebral hemorrhage. Docusate sodium is a stool softener, which helps prevent straining.
Straining would raise intracranial pressure.

191) A nurse is caring for a client with Parkinson’s disease who is taking benztropine mesylate
(Cogentin) daily. The nurse assesses the client for side effects of this medication and specifically
monitors:

Options:

A) Pupil response
B) Skin temperature

C) Intake and output

D) Prothrombin time

Correct Answer is: C

Explanation : Urinary retention is a side effect of benztropine mesylate. The nurse needs to
observe for dysuria, distened abdomen, voiding in small amounts, and overflow incontinence.
Options 1, 2, and 4 are not side effects of this medication.

192) A nurse is preparing to assess a client who was admitted to the hospital with a diagnosis of
trigeminal neuralgia (Tic Douloureux). On review of the client’s record, the nurse would expect
to note that the client experiences:

Options:

A) Chronic, intermittent pain in the seventh cranial nerve.

B) Abrupt onset of pain in the fifth cranial nerve

C) Bilateral pain in the sixth cranial nerve

D) Unilateral pain in the sixth cranial nerve.

Correct Answer is: B

Explanation : Trigeminal neuralgia is a chronic syndrome characterized by an abrupt onset of


pain. It involves one or more divisions of the trigeminal nerve or cranial nerve V. options 1, 3,
and 4 are incorrect.

193) A nurse is providing instructions to the spouse of a client who is taking tacrine (Cognex) for
the management of moderate dementia associated with Alzhemer’s disease. The nurse tells the
spouse:

Options:

A) If flu-like symptoms occur, it is necessary to notify the physician immediately

B) If a dose is missed, double up on the next dose


C) If a change in the color of the stools occurs, notify the physician

D) Do not administer the medication with food

Correct Answer is: C

Explanation : Tacrine (Cognex) may be administered between meals on an empty stomach, and if
gastrointestinal upset occurs, it may be administered with meals. Flu-like symptoms without
fever is a frequent side effect that may occur with the use of the medication. The client or
spouse should never be instructed to double the dose of any medication if it was missed, and
the client and caregiver are instructed to notify the physician if nausea, vomiting, diarrhea, rash,
jaundice, or changes in the color of the stool occur. This may be indicative of the potential
occurrence of hepatitis.

194) A client with refractory myasthenia gravis is told by the physician that plasmapheresis
therapy is indicated. After the physician leaves the room, the client asks the nurse to repeat the
physician’s reason for ordering this treatment. The nurse tells the client that this therapy will
most likely improve which of the client’s symptoms?

Options:

A) Urinary incontinence

B) Pins and needles sensation in the legs

C) Double vision

D) Difficulty breathing

Correct Answer is: D

Explanation : Plasmapheresis is a process that separates the plasma from the blood elements, so
that plasma proteins that contain antibodies can be removed. It is used as an adjunct therapy in
myasthenia gravis and may give temporary relief to the clients with actual or impending
respiratory failure. Usually three to five treatments are required. This therapy is not indicated for
the reasons listed in options 1, 2, and 3.

195) A client is admitted to the hospital in myasthenic crisis. The nurse questions the family
about the occurrence of which precipitating factor for this event?
Options:

A) Not taking prescribed medication

B) taking excess prescribed medication

C) Getting more sleep than usual

D) A decrease in food intake recently

Correct Answer is: A

Explanation : Myasthnic crisis is often caused by undermedication and responds to the


administration of cholinergic medications such as neostigmine (Prostigmin) and pyridostigmine
(Mestinon). Cholinergic crisis (the opposite problem) is caused by excess medication and
responds to withholding of medications. Change in diet and increased sleep are not precipitation
factors. However, overexertion and overeating could possibly trigger myasthenic crisis.

196) A client with trigeminal neuralgia asks the nurse what can be done to minimize the
episodes of pain. The nurse’s response is based on an understanding that the symptoms can be
triggered by:

Options:

A) Infection or stress

B) Excessive watering of the eyes or nasal stuffiness

C) Sensations of pressure or extreme temperature

D) Hypoglycemia and fatigue

Correct Answer is: C

Explanation : the paroxysms of pain that accompany this neuralgia are triggered by stimulation
of the terminal branches of the trigeminal nerve. Symptoms can be triggered by pressure form
washing the face, brushing the teeth, shaving, eating, and drinking. Symptoms can also be
triggered by thermal stimuli such as a draft of cold air. The items listed in the other options do
not trigger the spasm.
197) A client has been diagnosed with Bell’s palsy. The nurse assesses the client to see if which
signs and symptoms are visible?

Options:

A) Speech difficulties and one-sided facial droop

B) Twitching of one side of the face and ruddy cheeks

C) Eye paralysis and ptosis of the eyelid

D) Fixed pupil and an elevated eyelid on one side

Correct Answer is: A

Explanation : Bell’s palsy is a one-sided facial paralysis resulting from compression of the facial
nerve (CN VII). There is facial droop from paralysis of the facial muscles, increased lacrimation,
painful sensations in the eye, face, or behind the ear, and speech or chewing difficulties. The
other items listed are not associated with this disorder.

198) A nurse is admitting a client to the hospital who has a diagnosis of guillain-barre syndrome.
After ensuring that the client’s vital signs are stable and that the client is comfortable in bed, the
nurse asks a family member if the client has recently had:

Options:

A) A respiratory or gastrointestinal (GI) infection

B) Meningitis

C) A back injury or spinal cord trauma

D) Seizures or head trauma

Correct Answer is: A

Explanation : Guillain-Barre syndrome is a clinical syndrome of unknown origin that involves


cranial and peripheral nerves. Many clients report a history of respiratory or GI infection in the 1
to 4 week before the onset of neurological deficits. Occassionally, it has been triggered by
vaccination or surgery. The other options are not associated with an incidence of this syndrome.
199) A nurse is giving instructions to a client with peptic ulcer disease about symptom
management. The nurse tells the client to:

Options:

A) Eat slowly and chew food thoroughly

B) Eat large meals to absorb gastric acid

C) Limit the intake of water

D) Use acetylsalicylic acid (aspirin) to relieve gastric pain

Correct Answer is: A

Explanation : The client with a peptic ulcer is taught to eat smaller, more frequent meals to help
keep the gastric secretions neutralized. The client should eat slowly and chew thoroughly to
prevent excess gastric acid secretion. The client would consume fluids of 6 to 8 glasses of water
per day to dilute gastric acid. The use of aspirin is avoided, because it is irritating to gastric
mucosa.

200) A client with a hiatal hernia asks the nurse about fluids that are safe to drink and that will
not irritate the gastric mucosa. The nurse tells the client to drink.

Options:

A) Tomato juice

B) Orange juice

C) Grapefruit juice

D) Apple juice

Correct Answer is: D

Explanation : Substances that are irritating to the client with hiatal hernia include tomato
products the circus fruits, which should be avoided. Because caffeine stimulates gastric acid
secretion, beverages that contain caffeine, such as coffee, tea, cola, and cocoa, are also
eliminated from the diet.

You might also like